UNMC health assessment exam 4 practice questions

Pataasin ang iyong marka sa homework at exams ngayon gamit ang Quizwiz!

A 70-year-old man presents with the following symptoms: straining to void, nocturia, dribbling, and hesitancy when voiding. These signs are consistent with what condition? a. Benign prostatic hypertrophy (BPH) b. Prostatitis c. Testicular cancer d. Phimosis

A ( Rationale: As men age, fibromuscular structures of the prostate gland atrophy and are gradually replaced by collagen, which enlarges the gland. Consequences include nocturia, dribbling, and hesitancy when voiding.)

The nurse notes an irregular radial pulse in a patient. Further evaluation includes assessing a. for a pulse deficit. b. the carotid pulse. c. for diminished peripheral circulation. d. the brachial pulse.

A ( Rationale: Assessing for a pulse deficit provides an indirect evaluation of the heart's ability to eject enough blood to produce a peripheral pulse. When a pulse deficit is present, the radial pulse is less than the apical pulse.)

The patient with a head injury and increasing ICP is likely to have which assessment findings? a. Decreased LOC and sluggish pupil b. Left-sided weakness and facial droop c. Right ptosis and right-sided loss of vision d. Dilated left pupil and receptive aphasia

A ( Rationale: Because increasing intracranial pressure is a global process, the findings are more general and less specific. Findings localized to the left or right side are more commonly associated with specific areas of the brain, as with a stroke.)

A patient with a tympanic abdomen complains of pain in the RUQ. Which sign would the nurse expect to be positive? a. Murphy sign b. Psoas sign c. Rovsing sign d. Obturator sign

A ( Rationale: The Murphy sign tests for gallbladder pain. The other signs test for peritoneal irritation in the lower quadrants.)

A Pap smear is recommended to screen for what condition? a. Cervical cancer b. Ovarian cancer c. Endometrial cancer d. Vaginal cancer

A (Rationale: A Pap smear is a screening tool for detecting precancerous or cancerous cells of the cervix.)

The nurse palpates a fine, round, mobile, nontender nodule and suspects that it is a. a fibroadenoma. b. a cyst. c. a fibrocystic breast change. d. breast cancer.

A (Rationale: A cyst is soft to firm, often tender, round, and mobile. Fibrocystic breast changes feel nodular and ropelike. Breast cancer is irregular, firm, and fixed.)

The nurse auscultates a medium-loud whooshing sound that softens between S1 and S2. The nurse documents this finding as which of the following? a. Grade III systolic murmur b. Grade I systolic murmur c. Grade V diastolic murmur d. Grade II diastolic murmur

A (Rationale: A medium loud murmur is graded III or IV on a I to VI scale. Murmurs between S1 and S2 are systolic; those between S2 and S1 are diastolic.)

When assessing a child, the nurse makes the following adaptation to the usual techniques: a. A pediatric stethoscope is used for better contact. b. The child is seated away from the parent. c. The room is full of toys for play. d. The child is undressed, including the diaper.

A (Rationale: A pediatric stethoscope is smaller than the adult-sized one, allowing for the full diaphragm to be sealed on the patient's skin. The parent may wish to hold the child for security and comfort. If the room is full of toys, the child may prefer to play and be hesitant to be examined. The child is kept covered as much as possible to avoid chilling; when clothes are removed, the diaper usually partly covers the genitals to prevent the child from involuntarily urinating on the examiner)

A patient reports that a previous right hip replacement is suddenly painful. Which hip assessment technique should you omit? a. Adduction b. Hyperextension c. Extension d. Circumduction

A (Rationale: Adduction of the hip may cause the artificial hip to dislocate. The other activities are not restricted.)

A patient is anxious, dyspneic, and pale and uses accessory muscles to breathe. Vital signs are temperature 37°C (98.6°F), pulse 126 beats/min, respirations 40 breaths/min, and BP 122/74 mm Hg. The type of assessment that the nurse would perform is a(n) a. emergent assessment. b. general survey. c. health history. d. objective assessment.

A (Rationale: An emergent assessment focuses on data related to the problem, so that interventions can be implemented early and the problem can be resolved. Both subjective and objective data are gathered in the acute assessment.)

The nurse auscultates an extra sound on a patient 1 week after an MI. It is immediately after S2 and is heard best at the apex. Which of the following does the nurse suspect? a. S3 gallop b. S4 gallop c. Systolic ejection click Split S2

A (Rationale: Because of the clinical situation, the nurse is concerned about the pump function with loss of muscle from the MI. The S3 gallop results from a forceful atrial contraction during presystole that ejects blood into a ventricle that cannot expand further. The S4 gallop immediately precedes S1, and the S3 follows S2.)

While assessing a patient, the nurse finds a palpable lymph node in the left supraclavicular region. Which of the following should be the next action? a. Recognize that it is not common to palpate lymph nodes in this region and they must be carefully evaluated. b. Recognize that enlarged lymph nodes in this area indicate sinus inflammation. c. Recognize that this is a common area for lymph nodes to be enlarged with minor infections. d. Recognize that a palpable lymph node in this region is always indicative of malignancy.

A (Rationale: Cancers of the lung, breast, and abdomen may metastasize to the lymph nodes and be first accessible during clinical assessment in the supraclavicular region.)

Gynecomastia may occur in an older male secondary to a. testosterone deficiency. b. lymphatic engorgement. c. trauma. d. decreased activity level.

A (Rationale: Changes in testosterone levels promote breast growth. Lymphatic engorgement does not naturally accompany aging. Trauma may cause inflammation but not gynecomastia. Decreased activity level may occur with aging, but it does not affect the breast tissue.)

A 45-year-old man has been admitted to the hospital with suspicion of PE. Which of the following symptoms should the nurse report to the primary health practitioner immediately? a. Chest pain b. Shortness of breath c. Respirations 20 breaths/min d. Productive cough

A (Rationale: Chest pain is assumed to be heart pain and must be evaluated immediately. Ischemic heart pain, such as with a myocardial infarction, must be ruled out before considering another diagnosis.)

When the nurse listens to S1 in the mitral and tricuspid areas, the expected finding is a. S1 greater than S2. b. S1 is equal to S2. c. S2 greater than S1. d. No S1 is heard

A (Rationale: Closure of the mitral and tricuspid valves at the beginning of systole produces the S1. This closure prevents backflow of blood from the ventricles into the atria. S1 is loudest over these valves, located in the 5th left ICS at the sternal border (tricuspid) and the 5th left ICS at the MCL (mitral).)

During routine physical examination of a 20-year-old woman, the nurse notes a septal perforation. This finding may be significant for which of the following causes? a. Illicit drug use b. Nose picking c. Nasal trauma

A (Rationale: Cocaine and inhaled substances irritate the nose and may cause perforation.)

Which of the following are advantages of the electronic medical record? Select all that apply. a. Nurses can enter data by checking boxes and adding free full text. b. It is economical and easy to learn and implement. c. It allows primary care providers to directly order into the computer. d. It cannot be used as a legal document in case of a lawsuit.

A (Rationale: Computerized records usually have boxes to click and choices to make so that nurses do not have to write the whole assessment each time. They also have room for adding free text. Computerization ensures that all entries are legible and time dated. It minimizes compliance issues because programs will not let nurses enter data until they have completed all required fields. This ensures a more complete assessment. Although implementing a computerized system is expensive and requires much planning and education, such systems significantly increase patient safety. Computerized provider order entry (CPOE) allows providers to enter all orders directly into the computer, electronically communicating orders to the laboratory, pharmacy, and nursing unit. Although there is no hard copy, the eMAR is still considered the legal record.)

Peau d'orange appearance is highly suggestive of which of the following? a. Breast cancer b. Gynecomastia c. Papillomas d. Colostrum

A (Rationale: Enlargement of the breasts (B) is common in teenage boys and elderly men. C is small tumors of the subareolar ducts. D is clear, milky white fluid that precedes milk production. Peau d'orange (i.e., orange peel) appearance is caused by breast edema from blocked lymph drainage and indicates advanced cancer.)

One of the guests at a health promotion fair asks the nurse, "What is the greatest killer of women?" The nurse knows by current evidence that it is a. cardiovascular disease. b. lung cancer. c. breast cancer. d. osteoporosis.

A (Rationale: Heart disease is the number one killer of women.)

During history taking, a patient reports cramping in his calf when walking a few blocks. He states that it goes away when he sits down for a few minutes. How would the nurse document this symptom? a. Intermittent claudication b. Rest pain c. Poikilothermia d. Venous stasis

A (Rationale: Intermittent claudication is the appropriate terminology when a patient has pain that comes on with activity or exercise and goes away with rest.)

The nurse recognizes that the 60-year-old patient may have difficulty reading fine print because of a. the loss of accommodation. b. anisocoria. c. amblyopia. d. asthenopia.

A (Rationale: Loss of the ability to accommodate (contract the pupil for near vision) begins at around age 40 years, when many patients start using reading glasses. Unequal pupils usually are from a defect in efferent nervous pathways controlling the oculomotor nerve. Amblyopia (lazy eye) is used to describe a condition in which vision in one eye is reduced because the eye and brain do not work together correctly. Asthenopia (eye strain) develops after reading, computer work, or other visually tedious tasks.)

Which of the following assessment tasks can you appropriately delegate to an unlicensed care provider? a. Height, weight, and vital signs b. Active and passive c. ROM History of current complaint d. Muscle strength

A (Rationale: Nurses frequently delegate the taking of height, weight, and vital signs to unlicensed care providers. The other items are parts of assessment that cannot be delegated to unlicensed personnel.)

Which of the following patients is most likely to have hearing loss? a. Caucasian man older than 70 years of age b. Hispanic woman older than 50 years of age c. Asian man younger than 30 years of age d. African American girl younger than 10 years of age

A (Rationale: Older adult men of European descent have the highest incidence, which may be related to previous occupational exposure.)

A nurse observes a skin lesion with well-defined borders on the upper left thigh. It is 1.5 cm in diameter, flat, hypopigmented, and nonpalpable. What is the correct terminology for this lesion? a. Patch b. Plaque c. Papule d. Macule

A (Rationale: Patches are nonpalpable, defined lesions larger than 1.0 cm. Macules have the same characteristics of patches but are less than 1.0 cm. Papules are solid, raised, palpable lesions less than 1.0 cm. Plaques are papules larger than 1.0 cm.)

A 50-year-old patient is seen in the clinic for an annual physical examination and screening. The patient has no known health problems. This type of care is referred to as a. primary prevention. b. promotion prevention. c. tertiary prevention. d. healthy prevention.

A (Rationale: Primary prevention is screening and teaching that occur before disease. Secondary prevention focuses on preventing problems once disease is detected. Tertiary prevention addresses reducing complications from known disease.)

Risk factors for nose, sinus, mouth, and throat problems include a. topical decongestant use, smoking, and allergies. b. smoking, allergies, and high blood cholesterol. c. allergies, high blood cholesterol, and topical decongestant use. d. high blood cholesterol, topical decongestant use, and smoking.

A (Rationale: Risk factors specific to this area include topical decongestant use, smoking, inhaling substances and chemicals, allergies, and dust exposure.)

A 24-year-old patient reports an itchy red rash under her breasts. Examination reveals large, reddened, moist patches under both breasts in the skin folds. Several smaller, raised, red lesions surround the edges of the larger patch. What is the correct terminology for the distribution pattern of these smaller lesions? a. Satellite b. Discrete c. Confluent d. Zosteriform

A (Rationale: Single lesions in close proximity to a larger lesion are termed satellite lesions. Discrete distribution identifies lesions that are totally separate from one another. Confluent lesions are several lesions that have merged together, and zosteriform distribution identifies lesions, which follow a dermatomal pathway.)

Standard precautions a. are used on every patient because it is not always known whether a patient is infected. b. state that hand gel is used for infection with Clostridium difficile. c. include the use of gowns, gloves, and masks with all patients. d. recognize that transmission-based precautions are common.

A (Rationale: Standard precautions are used with every patient to prevent exposure to potential viruses, bacteria, or fungi. Hand gel is ineffective against C. difficile. Gowns, gloves, and masks are used only when there is potential contact with body secretions. Transmission-based precautions, including droplet, airborne, or contact precautions, are used with selected groups of patients who have identified infections.)

An unconscious 22-year-old man arrives at the hospital after experimenting with hallucinogenic substances. His vital signs are temperature 37.2°C, orally; pulse 142 beats/min; respirations 20 breaths/min; BP 100/64 mm Hg. The patient is experiencing a. tachycardia. b. eupnea. c. auscultatory gap. d. asystole.

A (Rationale: Tachycardia is a heart rate greater than 100 beats/min in an adult.)

A 20-year-old male patient presents with scrotal pain. A suspected diagnosis that requires immediate referral is a. testicular torsion. b. hydrocele. c. epididymitis. d. inguinal hernia.

A (Rationale: Testicular torsion requires immediate surgical intervention to prevent strangulation of the testicle.)

The nurse gathers subjective data related to the history of the present problem. The following items are included: a. Onset, location, duration, character, aggravating/associated factors, relieving factors, temporal factors, severity b. Asymmetry, borders, color, diameter c. Heart rate, respiratory effort, response, color d. Eye opening, verbal response, motor response

A (Rationale: The OLDCARTS mnemonic may be used to describe history of the present problem. B is the warning signs for skin cancer; C is elements of the Apgar score; D is elements in the Glasgow Coma Scale.)

Which of the following are appropriate interventions for the patient who is at risk for ear infection? Select all that apply. a. Be current on immunizations. b. Avoid secondhand smoke. c. Clean only external ear. d. Have audiogram yearly.

A (Rationale: The audiogram is recommended if screening reveals problems.)

While evaluating the inguinal lymph nodes of a patient, the nurse palpates a 1-cm (about ½-in.) soft and freely movable node. What action should the nurse take next? a. Nothing—this finding is normal. b. Refer this patient to a specialist. c. Immediately check the patient's dorsalis pedis pulse. d. Refer the patient for immediate management of a life-threatening condition.

A (Rationale: The documentation reflects findings consistent with a normal lymph node.)

The function of the ear is for a. hearing and equilibrium. b. equilibrium and perforations. c. perforations and balance. d. balance and equilibrium.

A (Rationale: The ear has two primary functions: hearing and equilibrium. Perforations of the tympanic membrane (TM) are abnormal. Equilibrium means balance.)

Which assessment technique best confirms splenic enlargement? a. Deep palpation under the left costal margin b. Fist percussion of the spleen with the patient in a sitting position c. Deep palpation over the RUQ with the patient lying on the right side d. Percussion along the left MAL spleen and gentle palpation

A (Rationale: The epigastric region is located above the umbilicus and straddles the midline between the right and left upper quadrants.)

Which of the following statements describes the cardiovascular system most accurately? a. It is a double pump circulating blood out to the lungs and body. b. It has a heart with six chambers, great vessels, and valves. c. It includes concepts of precontractility, postcontractility, and load. d. It functions with a conduction system that starts in the ventricles.

A (Rationale: The heart is a double pump with four chambers, four valves, and a conduction system that has a pacemaker originating in the atrium (SA node). Concepts of preload, afterload, and contractility are used when considering the effectiveness of the pumping.)

A 26-year-old man was in a motor vehicle accident and suffered a complete spinal cord injury to L3. The nurse assesses the patient for loss of motor function in the: a. legs b. abdomen c. chest d. arms

A (Rationale: The level of injury in the spinal cord correlates with innervation on the skin according to the level of the dermatome. Innervation of the arm roughly correlates with C5 to T1. Innervation of the chest correlates with T1 to T8. Innervation of the abdomen corresponds to T9 to T12. Innervation of the legs corresponds to L1 to S1.)

The inner ear a. contains the malleus, incus, and stapes. b. conducts sound waves to the external ear. c. translates sound to the nerves and brainstem. d. provides the body with proprioception.

A (Rationale: The malleus, incus, and stapes are in the middle ear. They conduct sound waves to the inner ear. The semicircular canals and vestibule provide the body with proprioception and equilibrium.)

Which organs or body areas does the nurse auscultate as part of the admitting assessment? a. Heart, lungs, and abdomen b. Kidneys, bladder, and ureters c. Abdomen, flank, and groin d. Neck, jaw, and clavicle

A (Rationale: The nurse auscultates heart, breath, and abdominal sounds as part of the complete assessment. All these involve movement, which generates sounds.)

Children are usually brought for health care visits by a parent. At about what age should you begin to question the child, rather than the parent, regarding presenting symptoms? a. 5 years of age b. 7 years of age c. 9 years of age d. 11 years of age

A (Rationale: The nurse should begin to ask the child direct questions early to encourage self-care and to assist in establishing rapport. Of course, the information received from a child of any of the listed ages would be confirmed, refined, or denied by the parent.)

The lymph nodes that lie in front of the mastoid bone are the a. preauricular nodes. b. occipital nodes. c. superficial cervical nodes. d. supraclavicular nodes.

A (Rationale: The preauricular are, as the name implies, in front of (or pre-) the ear (auricle). Occipital nodes are at the base of the skull posteriorly. Cervical nodes are in the neck, and supraclavicular are above the clavicle.)

A patient develops a sudden onset of acute chest pain. In addition to a complete description of the symptoms, what objective assessment is a priority? a. Pulse, blood pressure, peripheral pulses b. Heart sounds, rate, and rhythm c. Circulation, sensation, and movement d. Murmurs, rubs, and gallops

A (Rationale: The primary concern is the effect of cardiac ischemia on the patient. To know how well the heart is circulating blood, the nurse assesses blood pressure and peripheral pulses. Findings provide information about cardiac output. B and C provide data about the heart, but not about its effectiveness; D is assessment of the periphery only.)

An 83-year-old woman is undergoing a routine physical examination. Which of the following assessment findings would the nurse consider an expected age-related variation? a. Thinning of the skin b. Increased skin turgor c. Hypopigmented flat macules and patches over sun-exposed areas d. Multiple purplish bruises on the arms and legs

A (Rationale: The skin layers thin with aging, resulting in decreased skin turgor. Thinned skin is subject to increased trauma from shearing or friction, which increases the risk for purpuric lesions. Nevertheless, such lesions are not a normal variant of aging skin. Hyperpigmented macules and papules (commonly seborrheic keratoses) are present on sun-damaged skin.)

When percussing the abdomen, the nurse notices a dullness at the anterior right costal margin at the right midclavicular line. Which organ is most likely involved? a. Liver b. Spleen c. Sigmoid colon Kidney

A (Rationale: The spleen is normally found in the 9th to 11th left intercostal space (ICS) in the left midaxillary line (MAL). The colon is in the lower quadrants of the abdomen. The kidney is located in the posterior flank, in the lower rib cage. It is percussed for tenderness and is not always palpable.)

The nurse knows that the floor of the mouth is highly vascular and therefore a good location for which of the following? a. Absorption of sublingual medications b. Identification of malignancy in the pharyngeal fossa c. Infection with streptococcus d. Aspiration, even if the gag reflex is present

A (Rationale: The sublingual palate is a good location for taking oral temperatures and for the absorption of sublingual medications.)

While examining the patient's neck, the nurse finds the trachea midline but has difficulty palpating the thyroid. What action would the nurse take next? a. Document this finding as normal. b. Tell the patient that this finding is unexpected. c. Report to the physician a suspicion of a slow-growing goiter. d. Look for signs of hypothyroidism.

A (Rationale: The thyroid gland is often not palpable. With no signs or symptoms of hypothyroidism or hyperthyroidism, a nonpalpable thyroid would be a normal finding.)

A young male presents for a sports physical examination. In addition to examining for hernias, it would be appropriate for you to do which of the following? a. Teach testicular self-examination. b. Evaluate for urinary retention. c. Examine for prostate cancer. d. Draw blood to measure prostatic surface antigen.

A (Rationale: This age group is at high risk for testicular cancer; prostate cancer usually occurs later in life.)

Which phase of the nursing process is most foundational for delivery of care? a. Assessment b. Planning c. Diagnosis d. Evaluation

A (Rationale: This assessment determines which diagnoses will be the focus of care, the interventions that will be initiated, and those that will be reevaluated. In this way, the assessments drive care, and the reassessments loop back into further assessments and revision of care planning.)

The most common format for the comprehensive admitting assessment in the hospitalized adult is the a. head to toe. b. body systems. c. functional framework. d. systems framework.

A (Rationale: This format is efficient and comfortable for the patient. Focused assessments are usually integrated.)

Which of the following teaching points related to eye health would the nurse emphasize? a. Always wear eye protection for occupational exposures. b. Eat a diet high in animal protein and dairy. c. Exercise five times a week for at least 20 minutes. d. Get at least 7 hours of sleep each night.

A (Rationale: Two health goals are to (1) increase the proportion of public and private schools that require use of appropriate head, face, eye, and mouth protection for students participating in school-sponsored physical activities and (2) reduce occupational eye injury and increase the use of appropriate personal protective eyewear in recreational activities and hazardous situations around the home.)

All formats of progress notes a. use the nursing process in some form to show nursing thinking. b. identify the patient outcomes or goals to evaluate. c. include head-to-toe assessment data for completeness. d. have a section for evaluation of care so that nurses may revise interventions.

A (Rationale: Types of progress notes include narrative, SOAP, PIE, and focus notes. Nurses arrange narrative notes, which are the most loosely organized, by time or topic; they also usually cluster data and include some interventions in these notes. SOAP, PIE, and focus notes allow nurses to cluster data and reflect the critical thinking and diagnostic reasoning they used to plan and evaluate care. Case notes, care plans, or care maps may include patient outcomes or goals. They usually include an evaluation section near the outcomes. Usually, the assessment information focuses on the problem, and nurses write the complete head-to-toe information on an assessment flow sheet.)

A patient presents to the clinic with erythematous vesicles on the face and chest. Some vesicles have broken open, revealing a moist, shallow, ulcerated surface; some have scabbed over. Which of the following infectious illnesses does the nurse suspect? a. Varicella b. Measles c. Roseola d. Herpes simplex

A (Rationale: Varicella (chicken pox) is a highly contagious infectious disease. It occurs most frequently in children. It is characterized by single to multiple erythematous vesicles anywhere on the body. As the disease progresses, the vesicles progress into shallow ulcers covered with scabs. Measles is a rash of macules and papules. Herpes simplex is generally localized to one area of the body and consists of grouped vesicles on an erythematous base. Roseola is a macular and papular rash.)

A 62-year-old woman comes to the clinic with an exacerbation of asthma. Which of the following findings indicate worsening status of her asthma? a. Increased wheezing b. Sustained rhonchi c. Decreased respirations d. Oxygen saturation 94%

A (Rationale: Wheezing is associated with the airway inflammation and narrowing that accompany asthma. Bronchial rhonchi indicate secretions in the airway such as pneumonia and are not expected with asthma. Increased respirations are expected with decreased oxygenation. Pulse oximetry less than 92% is cause for concern.)

Select all actions that are acceptable under the HIPAA Privacy Rule. a. Communicate report with the next nurse during change of shift. b. Communicate with the primary care provider about a patient's change in assessment. c. Consult in the hall with the instructor about the patient's abnormal findings. d. Describe patient assessment findings to a colleague in the cafeteria.

A,B (Rationale: The HIPAA Privacy Rule requires an agency to make reasonable efforts to limit use of, disclosure of, and requests for protected health information to the minimum necessary to accomplish the intended purpose. Because the purposes of A and B are for the benefit of the patient, these are acceptable. Consulting with the instructor is also appropriate, but the hall is an inappropriate location to do so. Talking with a colleague is also acceptable in the context of learning, such as a post conference. Elevators, cafeterias, and other public spaces are inappropriate locations because visitors and other patients may become anxious or fearful when overhearing details related to illness, procedures, and other health-related concerns.)

Which of the following interventions is a priority for patient safety during care? STA a. Use two patient identifiers such as name and date of birth. b. Provide documentation, medical terminology, and SBAR for verbal communication. c. Use alarms safely, especially to prevent harm to patients at risk for falls. d. Proceed with surgeries immediately with no time-out.

A,B,C (Rationale: All surgeries must have a "time-out" period to avoid wrong site surgeries and other complications.)

A fair-skinned, blonde, 18-year-old woman is at the clinic for a skin examination. She reports that she always turns red within 10 minutes of going outside. She is planning a trip to Mexico and wants to avoid getting sunburned. Which of the following would be included in the teaching? (Select all that apply) a. Excessive exposure to UVA and UVB rays increases risk of sunburn and skin cancer. b. Apply a sunscreen or sunblock at least 15 to 30 minutes before sun exposure. c. Avoid sun exposure between 10 AM and 4 PM to reduce UVA and UVB exposure. d. A mild sunburn is acceptable in a fair-skinned blonde person.

A,B,C (Rationale: Teaching the patient about the harmful effects of UVA and UVB exposure will help her understand the importance of sun protection. Sunscreens or sunblocks applied in time for the skin to fully absorb them afford the best protection. Avoiding the sun during the midday decreases exposure to intense and harmful UVA and UVB rays.)

Which of the following findings during the general survey may indicate a change in mental status? Select all that apply. a. Disheveled appearance b. Rapid speech c. Lethargy d. Asymmetrical movements

A,B,C (Rationale: The general survey provides valuable clues to the patient's overall status. Changes in appearance, speech, and alertness may indicate a change in mental status and require further evaluation. Asymmetrical movements may indicate a stroke and a specific change in neurological status.)

Select all of the documentation errors that are potentially high risk. a. Failure to document completely b. Inadequate admission assessment c. Charting in advance d. Bunch charting at the end of shift

A,B,C,D (Rationale: All are considered high-risk assessments for liability. In addition, falsifying patient records, failure to record changes in patient condition, failure to document that the nurse notified the primary care provider when the patient's condition changed, and failure to follow agency's standards or policies on documentation are high risk.)

Cues of hearing loss include which of the following? Choose all that are correct. a. Using a loud or monotonous voice b. Asking to repeat questions c. Concentrating on lip movement d. Leaning forward to hear

A,B,C,D (Rationale: All are correct. Additionally, the patient may lean forward with the "good" ear.)

When assessing a patient with atelectasis, what assessment findings are expected? Choose all that apply. a. Shortness of breath b. Decreased breath sounds c. Decreased oxygen saturation d. Increased tactile fremitus e. Hyperresonance

A,B,C,D (Rationale: With atelectasis, the lung tissue has collapsed, which leads to less tissue for oxygenation. Consequently, the oxygen saturation is low, breath sounds are decreased, and the patient is short of breath. Because the tissue is consolidated, tactile fremitus is increased. The percussion sound might be dull, not hyperresonant, as a result of consolidation.)

The nurse enters a patient's room and initiates a rapid response call based on which of the following assessments? Select all that apply. a. An acute change in oxygen saturation less than 90% despite oxygen administration b. An acute change in systolic blood pressure to less than 90 mm Hg or a sustained increase in diastolic blood pressure greater than 110 mm Hg c. New-onset chest pain d. An acutely cold, cyanotic, or pulseless extremity e. An acute change in pupillary response

A,B,C,D,E (Rationale: All of these are emergency situations and the nurse may need additional assistance to provide immediate interventions. Additional situations include unexplained lethargy, new seizure, temperature greater than 39.0°C (102.2°F), uncontrolled pain, acute change in urine output less than 50 ml (about 1¾ oz) over 4 hours and acute bleeding.)

Risk factors for hearing loss include which of the following? Choose all that are correct. a. Frequent ear infections b. Being current on immunizations c. Exposure to smoke d. Age less than 30 years

A,C (Rationale: Lack of immunizations and increased age are risk factors. Additionally, family history, some medications, loud noises, allergies, airplane travel, diving, and inappropriate cleaning of the ears are risk factors.)

Which actions will result in an inaccurate BP reading? Select all that apply. a. Obtaining a BP immediately after the patient has entered the room b. Using a BP cuff with a bladder length which is 80% of the arm circumference c. Asking the patient to hold out his or her arm above heart level d. Pumping the cuff 10 mm Hg above the palpated systolic BP

A,C,D (Rationale: Common errors in BP measurements can occur because of physical activity, incorrect cuff size, placing the arm above or below heart level, and failure to auscultate above an auscultatory gap. It is recommended to pump the cuff 20 to 30 mm Hg above the last sound.)

The ABCDEs of melanoma identification do not include a. Asymmetry: one half does not match the other half. b. Birthmark: café au lait spot that does not fade. c. Color: pigmentation is not uniform; there may be shades of tan, brown, and black as well as red, white, and blue. d. Diameter: greater than 6 mm. e. Evolving: any change in size, shape, color, elevation—or any new symptom such as bleeding, itching, or crusting.

B ( Rationale: The B in ABCDE stands for irregular border of the lesion.)

The nurse participates in bedside handoff reports, practices hourly rounding, introduces himself or herself, addresses the patient using the name he or she prefers, and paraphrases patient responses to verify understanding. These are interventions designed to a. improve patient safety. b. increase patient satisfaction. c. improve infection rates. d. increase efficient assessments.

B ( Rationale: Patient satisfaction has been linked to improved outcomes, and nurses are routinely involved in assessing this.)

When the nurse assesses a client with respiratory symptoms, which of the following complaints should be evaluated first? a. Chest soreness b. Dyspnea c. Cough d. Sputum

B ( Rationale: Shortness of breath is observed during the initial contact with the patient. These data assist in determining the acuity of the problem. Chest tenderness with palpation will be assessed during the history; sputum and lung sounds are assessed during the physical assessment.)

Use of the GCS provides relatively objective assessment of LOC. The three functions assessed are: a. pupil reaction, orientation, and sensation b. verbal response, eye opening, and motor response c. eye opening, motor response, and sensation d. verbal response, pupil reaction, and motor response

B ( Rationale: The Glasgow Coma Scale (GCS) does not include pupillary response and sensation. Abnormalities of pupil reaction are associated with altered consciousness but may also result from peripheral nerve injury. Sensation cannot be assessed accurately if the patient has any difficulty with communication.)

A 47-year-old woman states she is having vertigo and some difficulty with balance. The nurse should assess: a. accommodation b. the whisper test c. shoulder strength d. soft touch

B (Balance and equilibrium are associated with cranial nerve VIII. Performing a whisper test will evaluate hearing, also associated with CN VIII. Testing for accommodation evaluates CN III. Shoulder shrug assesses CN XI and soft touch assesses CN V.)

The nurse performs BP screening at the local community center. As part of the health promotion intervention, the nurse also discusses the following risk factors for stroke: a. Low BP, lack of exercise, and diet high in fat b. High BP, diet high in fat, and smoking c. Diet high in fat, smoking, and walking five times weekly d. Obesity, swimming five times weekly, high BP

B (Rationale: A health history of diabetes mellitus, carotid artery disease, atrial fibrillation, and sickle cell disease places a person at risk for neurovascular disease. Additionally, the lifestyle choices of smoking, high-fat diet, obesity, and physical inactivity increase the person's risk for stroke.)

A dorsalis pedis of +1/4 may indicate a. DVT. b. PAD. c. Raynaud disease. d. Lymphadenopathy.

B (Rationale: A weak pulse is most closely correlated with PAD.)

The patient is complaining of abdominal pain. What technique is used to form an overall impression? a. Auscultation b. Light palpation c. Direct percussion d. Deep palpation

B (Rationale: An overall impression of the abdomen is gained by lightly palpating for tenderness and firmness. Auscultation provides information about gastrointestinal motility. Percussion provides information about an air-filled versus a solid or fluid-filled cavity. Deep palpation is used to identify the location of organs, masses, or tumors.)

In the SBAR reporting format, which of the following would be an example of data found in the assessment? a. Mrs. Kelly's diagnosis is Stage II breast cancer. b. Mr. Imami's lung sounds are decreased. c. Ms. Choi needs to have a social work consult. d. Mr. Jones was admitted at 10:30 this morning.

B (Rationale: Assessment findings are subjective or objective data.)

Auscultation is one of the most important components of which body systems? a. Reproductive, neurological, integumentary b. Cardiovascular, pulmonary, gastrointestinal c. Pulmonary, gastrointestinal, neurological d. Gastrointestinal, neurological, reproductive

B (Rationale: Auscultation plays a minimal role in the reproductive, neurological, and integumentary systems. Auscultation of the heart provides information on rate, rhythm, extra sounds, and murmurs. Auscultation of the lungs provides information on the underlying sound and adventitious sounds, which relate to pathology in the alveoli and airways. Gastrointestinal sounds may be absent, hypoactive, or hyperactive.)

When assessing the lower extremities, it is critical that the examiner a. starts at the feet. b. compares side to side. c. evaluates the venous system and then the arterial system. d. starts at the femoral area.

B (Rationale: Bilateral comparison is essential for accurate assessment of the peripheral vascular system.)

When the nurse assesses a 78-year-old patient with pneumonia, what is the priority assessment? a. Breath sounds b. Airway patency c. Respiratory rate d. Percussion sounds

B (Rationale: Consider the ABCs. Airway always assumes priority.)

A 20-year-old Caucasian man complains of a mass in his left testicle. In addition to his age and race, what else is a risk factor for testicular cancer? a. Colon cancer in his mother b. Personal history of cryptorchidism c. UTI in the previous month d. Congenital hydrocele

B (Rationale: Cryptorchidism (undescended testicle at birth) is a risk factor for testicular cancer.)

A patient reports changes in bowel pattern. Which is the best question to determine normal bowel habits? a. How often do you have a bowel movement? b. What was your bowel pattern before you noticed the change? c. Is there a family history of irritable bowel syndrome?

B (Rationale: Determining the patient's bowel pattern before symptoms began is most valid in establishing the normal pattern.)

While the nurse performs formal patient assessment, assistive personnel often observe changes when obtaining vital signs or assisting patients with ADLs. When discussing care for a patient with back pain, the nurse should particularly alert the assistant to watch for: a. dizziness b. bowel/bladder incontinence c. difficulty swallowing d. arm weakness

B (Rationale: Dizziness and difficulty swallowing are potential signs of cerebral rather than spinal cord lesions. Arm weakness from spine problems would indicate cervical injury (with associated neck rather than back pain). Bowel and bladder incontinence can occur with spinal cord injury at any level.)

To correctly document that ROM in the fingers is full and active, you would write that the patient can a. perform rotation, lateral flexion, and hyperextension. b. make a fist, spread and close fingers, and do finger-thumb opposition. c. touch finger to own nose and to examiner's finger back and forth. d. perform supination, pronation, and lateral deviation.

B (Rationale: Finger movements are flexion, extension, abduction, and adduction. The fingers do not perform rotation or lateral flexion. Touching the finger to the nose is part of neurological assessment, not range-of-motion (ROM) testing. The wrist performs supination, pronation, and lateral deviation.)

The nurse practitioner is assessing a patient with frequent candidiasis. The test that the nurse will order for this patient is a. cultures for chlamydia. b. a blood test for glucose. c. a blood test for syphilis. d. a vaginal ultrasound.

B (Rationale: Frequent vaginal candidiasis can be a symptom of abnormal blood glucose levels.)

Which of the following interventions is most important to prevent nosocomial infections? a. Proper glove use b. Hand hygiene c. Appropriate draping d. Quiet environment

B (Rationale: Hand hygiene is the single most important intervention to prevent the spread of infection. Either handwashing or using hand gel between patients is acceptable.)

Which assessment findings would indicate that inhaled bronchodilators have been effective? a. Expiratory wheezing, O2 saturation 94%, pallor b. Vesicular breath sounds, O2 saturation 96%, pink c. Bronchial breath sounds, O2 saturation 100%, erythema d. Crackles, O2 saturation 90%, circumoral cyanosis

B (Rationale: If bronchodilators are effective, assessment findings would indicate adequate gas exchange. Abnormal findings include wheezing, low oxygen saturation, pallor, bronchial breath sounds, erythema, crackles, and cyanosis.)

The nurse assesses the following vital signs in a 78-year-old man: temperature 36.6°C, temporal; pulse 72 beats/min, regular, 2+; respirations 18 breaths/min, regular, no use of accessory muscles; BP 142/92 mm Hg. Which of the findings is abnormal? a. Pulse b. BP c. Respirations d. Temperature

B (Rationale: In older adults, both SBP and DBP increase due to increased stiffness of arterial walls. This finding is outside of the normal range. Temperature in the older adult tends to be at the lower range of normal.)

Which factor places an infant at greater risk than an adult for developing otitis media? a. Introduction of solid foods b. Eustachian tubes that are more horizontal (flat) than vertical and wide c. Immature cardiac sphincter d. Feeding in a semi-Fowler position

B (Rationale: Infants have horizontal (flat) and wide Eustachian tubes, which make them more prone to otitis media. Feeding in a semi-Fowler position helps decrease the risk of otitis media because the infant is not flat. Introduction of solids does not influence the incidence of otitis media. An immature cardiac sphincter causes vomiting, not otitis media.)

All the following may be symptoms of a child experiencing lead poisoning except a. irritability. b. cardiomegaly. c. headaches. d. abdominal pain.

B (Rationale: Lead poisoning may be associated with all the symptoms except cardiomegaly. Lead is a heavy metal, a neurotoxin, and is not cardiotoxic.)

Of the following changes, which is the earliest sign of progressing brain herniation that originates in the cerebral hemispheres? a. An enlarging pupil that is sluggishly reactive to light b. Altered mentation c. Widening pulse pressure with bradycardia d. Reflex posturing of extremities

B (Rationale: Mental status changes are the earliest (often initially subtle) indications of generalized hemispheric dysfunction and occur prior to the cranial nerve or brainstem compression required to produce the other listed signs.)

A patient with diabetes mellitus who closely monitors and controls her blood glucose level is very interested in preventing complications of her illness. Which teaching is a priority for the patient related to peripheral vascular circulation? a. How to count calories. b. How to assess her feet daily. c. Choosing complex carbohydrates d. Identifying venous insufficiency.

B (Rationale: Meticulous foot care is essential for patients with diabetes to prevent complications of ulcers.)

It is important to examine the upper outer quadrant of the breast because it is a. more prone to injury and calcifications. b. where most breast tumors develop. c. where most of the suspensory ligaments attach. d. the largest quadrant of the breast.

B (Rationale: Most tumors occur in this region called the tail of Spence.)

An adolescent male presents with complaints of nosebleeds. The nurse would further assess for a. hemangioma. b. nasal trauma. c. angiofibroma. d. cystic fibrosis.

B (Rationale: Nasal trauma is the most common cause of epistaxis in adolescents.)

The nurse has assessed the nose and documents expected findings as a. nose asymmetrical with clear drainage. b. nose symmetrical and midline. c. nose asymmetrical and proportional to facial features. d. nose symmetrical with yellow drainage.

B (Rationale: Normal documentation of the assessment of the nose would include findings such as symmetrical, midline, without drainage, and proportional to facial features.)

A patient with a history of cirrhosis tells the nurse that his abdomen seems to be getting larger and that he has gained 9.7 kg (20 lb) in the past 6 months. How will the nurse determine whether the abdominal enlargement is from accumulation of fluid or fat from the weight gain? a. Listen for a fluid wave b. Percuss the abdomen for shifting dullness c. Auscultate for lymph nodes d. Stroke the abdomen to elicit the abdominal reflex

B (Rationale: Percussing elicits a change from tympany to dullness when the abdomen is in its most dependent position. Fat remains static.)

A patient has several red, inflamed, superficial, palpable lesions containing a thickened yellowish substance. How would the nurse document this lesion? a. Papule b. Pustule c. Cyst d. Vesicle

B (Rationale: Pustules are palpable erythematous lesions containing pus or other infectious material. Papules are solid. Cysts can contain serous as well as infectious substances and extend into the deeper layers of skin. Vesicles are small, thin-roofed lesions containing clear serous fluid.)

Which of the following peripheral vascular diseases is not known to have a hereditary component? a. Lymphadenopathy b. Raynaud disease c. Abdominal aortic aneurysm d. PAD

B (Rationale: Raynaud disease has an unknown etiology.)

A patient in a nursing home was admitted with a diagnosis of dementia. He started a fire because he was cooking at home and forgot that he left a pan on the stove. The nursing diagnosis that is highest priority is: a. ineffective brain tissue perfusion b. risk for injury c. acute confusion d. impaired memory

B (Rationale: Safety assumes priority because of the risk for injury. Impaired memory is also a likely diagnosis because of his forgetfulness. No data exist about confusion, so that is an area that needs further assessment. Ineffective brain perfusion is associated more with a stroke.)

The nurse enters the patient's room for the first time during the shift and directly observes the patient for breathing, airway, skin color, dyspnea, and airway secretions. This assessment is performed as part of a/an a. acute assessment. b. safety assessment. c. continuing assessment. d. comprehensive assessment.

B (Rationale: Safety inspections are routinely performed when initially coming on to shift to make sure that the patient is not in acute distress and is in no immediate danger of falling or injuring himself or herself.)

Which of the following patients would require immediate nursing care? a. An 8-year-old girl with pink conjunctivae and drainage b. A 20-year-old man with sudden visual loss after playing football c. A 52-year-old woman with clouding of vision d. A 77-year-old man with loss of vision in his peripheral fields

B (Rationale: Sudden loss of vision is an emergency, especially following a potentially traumatic injury; this patient is at risk for retinal detachment. The 8-year-old girl may have conjunctivitis. The 52-year-old woman has symptoms of cataract, and the 77-year-old man has symptoms of macular degeneration.)

A public health nurse is performing annual vision screening for residents in senior housing. Which of the following charts would the nurse most likely be using? a. Jaegar chart b. Snellen chart c. Ishihara cards d. Confrontation cards

B (Rationale: The Allen chart is used to test near vision. Ishihara cards are used to test color blindness. Confrontation is a test for peripheral vision.)

An auscultatory gap is defined as a. a drop in the SBP of 15 mm Hg or more with position change. b. a period of silence heard between Korotkoff sounds. c. the difference between the apical and radial pulse. d. SBP minus the DBP.

B (Rationale: The auscultatory gap is the period of no Korotkoff sounds during auscultation of BP. It is caused by stiffening of the arterioles and is common in the elderly and in those with chronic disease.)

As soon as the child can stand, begin to measure the height in the upright position. a. True. Using the scale as soon as the child can stand next to it is fine. b. False. Measure the child standing starting between ages 2 and 3 years. c. It depends on when the child can stand independently. d. False. A child should always be measured in the recumbent position.

B (Rationale: The child should stand for height measurements between ages 2 to 3 years. Growth charts for children aged 2 to 20 years represent standing heights. Growth charts for children aged 0 to 3 years are recumbent heights.)

The cranial nerves involved with eye movement include a. II, V, and VII. b. III, IV, and VI. c. IV, V, and VIII. d. V, VI, and VII.

B (Rationale: The cranial nerves for the eye include II, III, IV, and VI. The optic nerve is in the CNS, whereas cranial nerves III, IV, and VI control the muscles of the eye.)

Mr. Brown was playing soccer and hurt his right knee. It appears swollen. What is the first assessment you should make? a. Palpate for crepitus in the knee. b. Compare the swollen knee with the other knee. c. Assess active ROM in the knee. d. Feel the knee for warmth.

B (Rationale: The first step is inspection. The first thing to do is to compare one knee with the other for symmetry. All the other answers are procedures for assessing joints, which may be indicated but do not represent the first step that the nurse should take.)

Which of the following organisms is associated with salpingitis? a. Trichinella spiralis b. Chlamydia trachomatis c. Candida albicans d. Condyloma acuminatum

B (Rationale: The most common organisms that cause salpingitis are Chlamydia trachomatis and Neisseria gonorrhoeae.)

A nursing diagnosis appropriate for a patient with ear problems is a. kinesthetic disturbed perception. b. disturbed sensory perception. c. sensory perception, gustatory. d. olfactory sensory perception.

B (Rationale: The nursing diagnosis is disturbed sensory perception. It is further defined as visual (eye), auditory (ear), kinesthetic (movement), gustatory (taste), tactile (touch), or olfactory (smell).)

Which of the following assessments is considered a basic care activity for the NCLEX licensing examination? a. Venous access devices, IV lines, invasive monitoring b. Personal hygiene habits, mobility routines c. Responses to procedures and treatments d. Vascular perfusion, hypotension, peripheral pulses

B (Rationale: The other items are considered physiological dimensions and assessments will be included in foundations and medical surgical courses.)

Which of the following descriptions is most consistent with a patient who has hypothyroidism? a. Slightly obese, perspiring female, who complains of feeling cold all the time and having diarrhea. b. Slightly obese female with periorbital edema, who complains of cold intolerance, brittle hair, dry skin. c. Thin, anxious-appearing female with exophthalmos and a rapid pulse and who complains of diarrhea. d. Thin, perspiring male with a deep hoarse voice, facial edema, a thick tongue, and reports of diarrhea.

B (Rationale: The patient with hypothyroidism would likely demonstrate clinical signs and symptoms of a low metabolic rate resulting from relative depletion of circulating thyroid hormone.)

The nurse assesses the response of the eye to light and documents normal findings as a. PEERLA. b. PERRLA. c. PERLLA. d. PERLAA.

B (Rationale: The pupils are equal, round, and reactive to light and accommodation. This is abbreviated as PERRLA.)

The nurse auscultates bronchovesicular breath sounds in the second ICS near the sternum. The nurse interprets this as a. a normal finding over the trachea. b. a normal finding over the bronchi. c. an abnormal finding over the lung. d. an abnormal finding over the trachea.

B (Rationale: The trachea bifurcates at the second intercostal space (ICS), and bronchovesicular sounds are expected. Bronchial breath sounds are auscultated over the trachea; vesicular breath sounds are heard over the lung fields.)

A patient complains of a soft, irregular mass on the left side of the scrotum he noticed while walking. The nurse palpates a mass that feels like "a bag of worms." These findings are consistent with which condition? a. Hydrocele b. Varicocele c. Spermatocele d. Epididymitis

B (Rationale: Varicocele is a condition caused by abnormal dilation and tortuosity of the veins along the spermatic cord, often on the left side. Upon palpation, the varicocele feels like a bag of worms.)

While reviewing laboratory values for thyroid function in an adult patient, the nurse sees that the TSH is elevated, and T3 and T4 are decreased. The nurse recognizes that these findings are indicative of a. normal thyroid function. b. hypothyroidism. c. hyperthyroidism. d. thyroid cancer.

B (Rationale: With hypothyroidism, TSH from the pituitary gland usually is increased. Because of decreased thyroid function, there is a decrease in circulating thyroid hormones as measured by T3 and T4 levels in the blood.)

Which of the following patients is at highest risk for osteoporosis? a. A young man, weight-lifter, who drinks beer three times a week, with a stable job b. A middle-age woman of lower socioeconomic status who is a heavy smoker and drinks alcohol six times a week c. A woman who works as a vice-president, takes a shot of vodka six times a week, and exercises regularly d. A retired man, non-smoker, who drinks alcohol socially

B (Rationale: Women of lower socioeconomic status are more likely to report limitations in activity and arthritis, obesity, and osteoporosis. Also, smoking increases the risk of developing fractures for both men and women. Alcohol use is associated with increased risk of osteoporosis because it raises parathyroid hormone levels, which causes calcium loss from bones.)

A patient who visits the clinic has the controllable risk factors of smoking, high-fat diet, overweight, decreased activity, and high blood pressure. What concept should the nurse use when performing patient teaching? a. Teach the patient the most serious information. b. Give the patient brochures to review before the next visit. c. Discuss risk factors that the patient is interested in modifying. d. Describe consequences of risk factors to motivate the patient.

C ( Rationale: Because multiple risk factors are apparent, the most effective strategy may be for the patient to identify those items that he or she would like to change. Presentation of many brochures is likely to overwhelm the patient; it is better to focus attention on one or two things that the patient is interested in modifying.)

Which of the following would you recognize as an unexpected finding while examining the male genitalia? a. Smegma is present on the uncircumcised patient. b. Testes are palpable and firm within the scrotal sac. c. You note an impulse at the tip of your finger during hernia examination. d. The urethral meatus has a slitlike opening central to the distal tip of the glans.

C ( Rationale: Indirect inguinal hernia presents with an impulse at the tip of the nurse's finger during hernia examination. All other answers represent normal findings.)

Which of the following is part of the upper gastrointestinal tract? a. Nasal septum b. Sinuses c. Throat d. Adenoids

C ( Rationale: The throat is part of the upper gastrointestinal tract. The other answers reflect parts of the upper respiratory tract.)

Which of the following is a normal ABI? a. 56 b. 87 c. 1.0 d. 24

C (Rationale: A normal ankle-brachial index (ABI) is 1.0 to 1.29. All other options represent problematic findings.)

The correct position in which to place the patient to palpate the breasts is a. left lateral position with arm over head. b. sitting forward with hands on hips. c. supine with arm over head. d. supine with arms at side.

C (Rationale: A pillow should also be placed under the patient's shoulder on the side being assessed. The patient should be supine for the examination. B indicates the position for inspection. Placing the arm over the head stretches the skin and makes palpation easier.)

Strategies for effective handoffs during change-of-shift report are to a. tape-record the report for efficiency. b. vary the format to individualize to the patient. c. allow an opportunity to ask and answer questions. d. put report in writing so that the next shift care provider can get right to work.

C (Rationale: A standardized format such as SBAR for handoffs ensures that nurses present all important information predictably and clearly. Face-to-face verbal updates of current status and historical data with interactive questioning are recommended for handoffs. It is best for nurses to perform handoffs in areas with limited interruptions, although finding such a location can be challenging during this busy time. Nurses use "readback" policies to ensure that both parties agree and comprehend high-risk procedures or medications. Verbal handoffs do not replace required written documentation because written documentation serves as the legal record. By reporting in person, nurses can cross-monitor the handoffs of others.)

The purpose of auditing charting is to a. enhance nurses' learning and understanding of complex clinical situations. b. identify staff members who document completely and counsel those who do not. c. determine whether staff members are providing and documenting standards of care. d. locate data in the chart the evening before a morning clinical visit.

C (Rationale: Agencies usually perform audits to look at systems, not individuals, and to determine whether staff members are meeting the standard of care. Accrediting agencies, such as The Joint Commission or Department of Health, audit charts to make sure that an agency is meeting state or federal standards. They also may review charts for financial reimbursement, especially Medicare or Medicaid. The charts are used for learning in grand rounds, in conferences, and for individual students of the health professions. Researchers also use charts to gather retrospective data.)

After receiving patient information from the previous shift nurse and gathering data from the chart, the nurse will assess a group of four patients. Which one will the nurse assess first? a. A 32-year-old man with an open wound who is receiving antibiotics b. A 66-year-old woman 2 days postoperatively following ankle surgery c. A 45-year-old man with HIV and Pneumocystis jiroveci pneumonia with dyspnea d. An 88-year-old woman with confusion who had a stroke 4 days ago

C (Rationale: Assessments are prioritized by airway, breathing, and circulation, so the patient with shortness of breath should be seen first. The patients receiving antibiotics and 2 days postoperative are stable. The elderly female with confusion would most likely be seen second because she may be at risk for injury or falls.)

The patient's muscle tone is hypertonic so the muscles are stiff and the movements are awkward. The nurse documents these findings as a. atony. b. tremors. c. spasticity. d. fasciculation.

C (Rationale: Atony is the lack of tone or strength, tremors are involuntary contractions of muscles, and fasciculation is involuntary twitching.)

The practitioner has decided to place a patient on isotretinoin for her acne problems. The nurse is preparing to counsel the patient. What is the most important information she needs to tell the patient? a. She needs to take the medication daily and avoid missing a dose. b. She should not take this medication with antibiotics. c. She needs to use two forms of birth control or abstain from sex 1 month before, during, and 1 month after taking this medication. d. She needs to take a weekly pregnancy test to make sure she has not gotten pregnant while on this medication.

C (Rationale: Because of the severe teratogenic effects of this medication, anyone of childbearing age must either abstain from sex or use at least two forms of birth control during treatment and for 1 month before and 1 month after treatment.)

A 23-year-old nulliparous woman is concerned that her breasts seem to change in size all month long and they are very tender around the time she has her period. The nurse should explain to her that a. nonpregnant women usually do not have these breast changes and this is cause for concern. b. breasts often change in response to stress, so it is important to assess her life stressors. c. cyclical breast changes are normal. d. breast changes normally occur during pregnancy and she should have a pregnancy test.

C (Rationale: Breasts often change throughout the menstrual cycle, with corresponding variations in hormonal levels.)

A clinical nurse is assessing a patient's knowledge and understanding of bone health and maintenance. Which of the following responses of the patient indicates adequate understanding to maintain musculoskeletal health? a. I will take calcium supplementation as prescribed and eat plenty of citrus fruits. b. I will expose myself to sunlight at least 1 hour daily and eat plenty of green, leafy vegetables. c. I will take calcium supplementation and vitamin D as prescribed. d. I will exercise daily and take vitamin E as prescribed.

C (Rationale: Calcium is essential for bone growth and remodeling. Vitamin D is essential for calcium absorption. Eating plenty of citrus fruits or increasing vitamin C intake will not assist in calcium absorption. Exposing to sunlight for at least an hour daily is not needed and is impractical. Weight-bearing exercises help build stronger bones, but vitamin E will not assist in calcium absorption.)

The nurse is admitting a 75-year-old man with a 50-year history of smoking 1 pack of cigarettes per day. Among the patient's concerns is his chronic shortness of breath. One nail finding that demonstrates chronic hypoxia is a. pitting. b. thickening and discoloration of the nail bed. c. clubbing. d. brittleness and cracking of the nails.

C (Rationale: Chronic hypoxia decreases oxygenation of the distal extremities. Associated clubbing changes will be evident.)

The best way to assess a client's respiration rate is by: a. Place a hand over the client's chest and count for 30 seconds b. Observe and count respirations for 30 seconds and multiply by two without mentioning that you are observing the respirations. c. Ask the client to breath normally for one minute. d. If respirations are irregular have the client rest for 10 minutes and then recount.

C (Rationale: Do not make the patient aware that you are assessing respirations. Increased awareness may alter normal respiratory pattern.)

What technique facilitates accurate auscultation? a. Earpieces of the stethoscope are positioned to point toward the back. b. The tubing of the stethoscope is long and dark in color. c. The chestpiece of the stethoscope is sealed against the skin. d. The diaphragm of the stethoscope is used for low-frequency sounds.

C (Rationale: Earpieces always point toward the front, following the same position as the nose. Tubing should be short and thick to optimize sound transmission. The chestpiece should be completely on the patient's skin to diminish transmission of room noise and to optimize sounds from the patient. The diaphragm is used for high-frequency sounds (e.g., bowel sounds); the bell is used for low-frequency sounds.)

When examining the scrotum of an adult Hispanic male, a normal finding is a. symmetrical scrotal sac with two movable testes. b. smooth, rubbery, saclike surface that is sensitive to gentle compression. c. asymmetrical sac with left side lower than right side. d. reddish colored skin that is darker than general body skin and has sebaceous cysts.

C (Rationale: Elevation of the affected testicle will usually lessen pain in epididymitis. All other choices usually present with testicular torsion.)

A patient with a history of kidney stones presents with complaints of pain, hematuria, and nausea with vomiting. What assessment technique will elicit kidney pain? a. Inspection with indirect lighting b. Iliopsoas muscle sign c. Indirect percussion for CVA tenderness d. Blumberg sign

C (Rationale: Fist percussion over the costovertebral angle (CVA) is the only technique listed that reflects a technique for assessing the kidney. The two specialty techniques are used to assess peritoneal inflammation.)

When performing an abdominal assessment, what is the correct sequence? a. Inspection, palpation, percussion, auscultation b. Palpation, percussion, inspection, auscultation c. Inspection, auscultation, percussion, palpation d. Auscultation, inspection, palpation, percussion

C (Rationale: For the abdomen, auscultation must be performed before percussion and palpation to prevent minimizing bowel sounds.)

When working with an older adult, what would the nurse emphasize as increased risks for the patient? a. Myopia and strabismus b. Blepharitis and chalazion c. Glaucoma and cataracts d. Exophthalmos and presbyopia

C (Rationale: Glaucoma, cataracts, and macular degeneration are all more common in the elderly. Myopia is nearsightedness; with strabismus, a person cannot align both eyes simultaneously under normal conditions (cross eyes). Blepharitis is inflammation of the margin of the eyelid; chalazion is a cyst in the eyelid. Exophthalmos is anterior protrusion of the eyeball out of the socket; presbyopia is believed to be caused by the loss of elasticity of the crystalline lens.)

Which of the following is an appropriate use of gloves? a. Gloves are worn during anticipated contact with intact skin. b. Gloves are removed when going from clean to contaminated areas. c. Gloves are worn during anticipated contact with body secretions. d. Gloves are removed when assessing the back of an incontinent patient.

C (Rationale: Health care providers should wear gloves to prevent exposure when they are at risk for coming into contact with body secretions of patients. The gloves protect patients by preventing nurses from transmitting infections from contaminated to cleaner areas. Generally, the area around the bed or examination table is considered most contaminated, whereas supply cupboards and computers are considered clean. Gloves should never be worn from the room into the hall.)

The nurse is taking a menstrual history. What would be an appropriate question to ask? a. Do you have any history of cancer in your family? b. Do you ever skip periods? c. Do you use condoms during intercourse? d. How many sexual partners have you had?

C (Rationale: In a menstrual history, the nurse asks information related only to menstrual function. History of cancer in relatives is part of family history. Questions related to sex or sexually transmitted infections (STIs) are asked later in the history, after the nurse has established a trusting relationship.)

When examining the breast of a 75-year-old woman, the nurse would expect to find which of the following? a. Enlarged axillary lymph nodes b. Multiple large firm lumps c. A granular feel to the breast tissue d. Pale areola

C (Rationale: In older women, secretion of estrogen and progesterone decreases, leading to atrophy of the glandular tissue and its replacement with fibrous connective tissue. This tissue feels granular. Axillary lymph nodes do not enlarge. Multiple large, firm lumps are a sign of benign breast disease (BBD), which occurs in patients aged 30 to 55 years. Areolae do not change in color.)

Tinnitus is described as a. inability to hear well. b. dizziness. c. ringing in the ear. d. ear pain.

C (Rationale: Inability to hear well is hearing loss, loss of equilibrium is dizziness, and ear pain is described as otalgia.)

Physical examination of a patient reveals an enlarged tonsillar node. Acutely infected nodes would be a. hard and nontender. b. fixed and soft. c. firm but movable and tender d. irregular and hard.

C (Rationale: Infected lymph nodes are usually tender. Fixed, hard, or irregular nodes should be further evaluated as a sign of possible cancer.)

A 22-year-old patient presents to the clinic with a large firm mass on her left earlobe. She had her ears pierced approximately 3 weeks ago. The mass began as a small bump and progressively enlarged to its current size of approximately 2.5 cm (1 in.) in diameter. It is not tender, reddened, or seeping any drainage. What is the term used to describe this secondary skin lesion? a. Crust b. Lichenification c. Keloid d. Scale

C (Rationale: Keloid is an excessive accumulation of fibrin tissue in response to wound healing. Lichenifications are exaggerated skin lines as a result of chronic irritation or scratching. Crust is a dried secretion from a primary lesion, and a scale results from excessive proliferation of the upper epidermal skin layers without normal shedding of dead cells.)

The nurse is assessing the nares to evaluate the site of epistaxis. The most common site of bleeding is which of the following? a. Ostiomeatal complex b. Nasal septum c. Kiesselbach plexus d. Woodruff plexus

C (Rationale: Kiesselbach plexus is a highly vascular area of the nose and a common site for bleeding.)

When doing an assessment of the spine of an older adult, you can expect to see which variation? a. Lordosis b. Torticollis c. Kyphosis d. Scoliosis

C (Rationale: Many older adults normally have an exaggerated forward curve of the thoracic spine, which may appear even more curved because of fat pad deposits.)

The nurse usually performs a complete physical examination with elements in the following order: a. Face, heart, legs, arms b. Head, abdomen, lungs, legs c. Eyes, heart, abdomen, legs d. Ears, back, lungs, arms

C (Rationale: Moving from head to toe is most efficient for the nurse and conserves energy for the patient. Subjective data are usually collected first. The most sensitive areas (e.g., genitals) may be deferred until last.)

Tympany is a percussion sound commonly located in the a. thorax. b. upper arm. c. abdomen. d. lower leg.

C (Rationale: Percussion sounds are hyperresonant (diseased lungs), resonant (normal lungs), tympanic (abdomen), dull (over organs), and flat (over bone).)

A history of smoking has an extremely significant role in the development of which of the following? a. Venous insufficiency b. DVT c. PAD d. Raynaud disease

C (Rationale: Smoking is one of the most devastating risk factors for peripheral arterial disease (PAD).)

Which of the following factors is the most significant risk factor for COPD? a. Increased age b. Immune suppression c. Tobacco smoking d. Occupational exposure

C (Rationale: Smoking is the most common cause of COPD. It is a risk that should be assessed; assistance with smoking cessation should be offered.)

A young adult marathoner reports of right foot third metatarsal pain (6/10) and swelling for more than 4 weeks. An x-ray was ordered, and it did not show abnormal findings. Which of the following imaging might the nurse expect the physician to order? a. Repeat x-ray b. CT scan c. MRI d. Nuclear scintigraphy

C (Rationale: Systematic reviews demonstrated that MRI has the highest specificity for diagnosing stress fractures and is followed by nuclear scintigraphy. Repeat x-ray imaging is not indicated and has the lowest specificity for detecting stress fractures. A CT scan is not the most appropriate imaging for stress fractures)

In a healthy patient, the myocardial cells in the ventricle depolarize and contract during a. prediastole. b. diastole. c. systole. d. postsystole.

C (Rationale: The SA node depolarizes and the atria contract, which appears as the P wave on an ECG. After a brief pause at the AV junction, the impulse travels through the bundle of His, bundle branches, and Purkinje fibers. The muscle cells depolarize and contract, appearing as the QRS complex on the ECG. The end result is ventricular systole and ejection of blood to the lungs and body, producing the heartbeat.)

Peripheral vision is evaluated by the nurse using the a. corneal light test. b. cover test. c. confrontation test. d. cardinal fields of gaze test.

C (Rationale: The corneal light reflex is a test for strabismus. The cover test is to assess the presence and amount of ocular deviation. The cardinal fields of gaze test is used to evaluate motor function in the eyes.)

The nurse assesses the child with purulent, unilateral nasal discharge. The nurse knows that the most likely causative factor is a. allergic rhinitis. b. choanal atresia. c. foreign body in nose. d. cystic fibrosis.

C (Rationale: The foreign body causes discharge; the most significant finding is that the drainage is unilateral. Most other processes involve both nares.)

When auscultating the abdomen, the nurse hears a bruit to the right of the midline slightly below the umbilicus. The nurse documents this finding as a bruit of which of the following? a. Right renal artery b. Right femoral artery c. Right iliac artery d. Abdominal aorta

C (Rationale: The iliac arteries are located to the left and right of the midline of the abdomen, below the umbilicus. The aorta is midline, the renal artery is above the umbilicus, and the femoral artery is located in the groin.)

The nurse assesses the neck vessels in the stable patient with heart failure to determine which of the following? a. The bilateral carotid pulse b. The presence of bruits c. The highest level of jugular venous pulsation d. The strength of the jugular veins

C (Rationale: The nurse looks for fluid volume overload in the patient with congestive heart failure (CHF). An elevated jugular venous pulsation reflects fluid volume overload in the right heart. The bilateral carotid pulse is never palpated, because doing so may obstruct the circulation to the brain and cause the patient to faint. Bruits are auscultated in the carotids for the presence of narrowing that may lead to stroke; the carotids, not the jugular veins, are also palpated for arterial pulse strength.)

The patient's radial pulse is weak and thready. The next action of the nurse is to a. transfer the patient to a critical care unit. b. notify the primary care provider. c. compare findings with previous findings and opposite extremity. d. assess vital signs every 15 minutes.

C (Rationale: The popliteal pulse is often difficult to palpate. Comparing to previous findings and to the opposite extremity can help determine if any acute changes have occurred.)

The seven Ps of an acute arterial occlusion include a. polythermia. b. popliteal pallor. c. poikilothermia. d. pitting edema.

C (Rationale: The seven "Ps" are pain, polar (poikilothermia), paresthesia, paralysis, pallor, pulse, and perfusion.)

The nurse can best evaluate the strength of the sternocleidomastoid muscle by having the patient a. clench his or her teeth during muscle palpation. b. bring his or her head to the chest. c. turn his or her head against resistance. d. extend his or her arms against resistance.

C (Rationale: The sternocleidomastoid muscles play an important role in turning the head from side to side. Asking the patient to turn the head against resistance is one way to determine that the strength of these muscles is symmetrical and equal.)

You obtain a blood pressure reading of 110/70 mm Hg (left arm) in a 5-year-old boy. What would you do about this blood pressure? a. Call the physician immediately. b. Bring the child back to the clinic two more times to ensure accuracy of the assessment. c. Determine the blood pressure percentile based on age, sex, and height percentiles. d. It is normal; nothing needs to be done.

C (Rationale: This blood pressure may be normal, but the nurse cannot tell until it is evaluated against the blood pressure norms for children in the same age range.)

When assessing hydration, the nurse will a. pinch a fold of skin on the medial aspect of the forearm and observe for recoil to normal. b. pinch a fold of skin on the abdomen and observe for recoil to normal. c. pinch a fold of skin just below the midpoint of one of the clavicles and allow the skin to recoil to normal. d. pinch a fold of skin on the head and allow for skin to recoil in children.

C (Rationale: To assess turgor in an adult, the most reliable method is to pinch a fold of skin on the anterior chest, release, and observe for the skin to promptly recoil to its original state.)

The nurse assesses a patient presenting with nausea, vomiting, and diarrhea. In performing the focused assessment, the nurse uses the following techniques: a. Auscultate lungs, auscultate heart, auscultate abdomen. b. Evaluate for dehydration, assess skin turgor, auscultate lungs. c. Auscultate abdomen, palpate abdomen, evaluate for dehydration. d. Palpate abdomen, percuss abdomen, auscultate heart.

C (Rationale: With nausea, vomiting, and diarrhea, concern arises about fluid volume deficit and the potential for dehydration, which would be noted with poor skin turgor. The lungs are not grouped with the symptoms. Auscultating the heart is an option to determine heart rate, but increases in heart rate can be evaluated when vital signs are collected. The abdomen needs to be auscultated to evaluate for suspected hyperactive sounds from the increased peristalsis.)

When teaching the breast self-examination, the nurse should inform the woman that it is best to perform the exam is which of the following times? Select all that apply. a. Just before the menstrual period b. Just after the menstrual period c. On the 4th to 7th days of the menstrual cycle d. On the 10th day of the menstrual cycle

C (Rationale: the breasts are least congested with the end of the menstrual period. In postmenopausal patients, the time of the month for the SBE is irrelevant, because breast size remains stable. For these patients, a day of the month that they will remember.)

Which of the following interventions are common in the hospitalized patient? Select all that apply. a. Assess pain every 8 hours and reassess 2 hours after interventions. b. Assess oxygen and vital signs every day. c. Assess Braden Scale and provide skin hygiene as needed every 8 hours. d. Perform focused assessments every shift and as needed. e. Perform safety assessments every day.

C,D (Rationale: Pain is reassessed 30 minutes after an intervention; vital signs are assessed every 4 to 8 hours routinely, and safety inspections are performed at the beginning of the shift and whenever needed.)

Which of the following clusters of symptoms are common in women preceding an MI? a. Chest pain, nausea, diaphoresis b. Weight gain, edema, nocturia c. Dizziness, palpitations, low pulse d. Fatigue, difficulty sleeping, dyspnea

D ( Rationale: Men typically have chest pain, nausea, and diaphoresis. Weight gain, edema, and nocturia are typical symptoms of CHF. Dizziness, palpitations, and low pulse are common with arrhythmias. Fatigue is a common presenting symptom in women.)

The correct position in which to place a healthy adult male client to examine the rectum and prostate is a. the left lateral Sims position with right knee flexed and left leg extended. b. the supine position with hips and legs flexed and feet positioned on the examining table. c. the modified knee-chest position with the patient prone and knees flexed under hips. d. standing and leaning over the examination table with chest and shoulders resting on the table.

D ( Rationale: Standing is preferred because it allows for visualization of the anus and palpation of the rectum. If the patient cannot stand, the Sims position (A) is used.)

The nurse is preparing the patient for her genital examination. What position will the nurse assist the patient into for a comfortable genital examination? a. Semi-Fowler b. Prone with her knees bent c. Supine with her knees bent d. Semi-lithotomy

D ( Rationale: The patient is placed in the semi-lithotomy position so that she has eye contact with the health care practitioner and can see what is going on.)

During a physical assessment, using the handle of the reflex hammer, you gently stroke the inner left thigh of the patient, which causes the ipsilateral testicle to rise. What superficial reflex is demonstrated? a. Abdominal reflex b. Babinski reflex c. Brachioradialis reflex d. Cremasteric reflex

D ( Rationale: The superficial cremasteric reflex is created by stroking the upper thigh, which causes the ipsilateral testicle to rise. Absence of this reflex is seen in association with disorders of the pyramidal tract above the level of the first vertebra.)

The chart states that a 62-year-old woman has had a stroke in the right parietal area of the brain. The nurse expects to note which of the following? a. Tremors on the left side of the face b. Tremors on the right side of the face c. Weakness in the right arm d. Weakness in the left arm

D ( Rationale: Weakness results from loss of motor function in the motor cortex of the brain. Tremors are associated with other diseases (e.g., Parkinson disease and multiple sclerosis). The deficit is on the opposite side of the body because the motor fibers cross, causing left-sided weakness.)

The nurse is inspecting the urethra and the Skene glands. She knows these are a part of what area? a. Mons pubis b. Vulva c. Posterior fourchette d. Vestibule

D ( Rationale: Within the vestibule in the upper middle area lies the urethra, with bilateral paraurethral Skene glands at the 7-o'clock and 5-o'clock positions, respectively. )

Which of the following symptoms would the nurse expect the patient to report as translucent specks that drift across the visual field? a. Blind spots b. Ptosis c. Halos d. Floaters

D (Rationale: A blind spot may be related to a problem of the optic nerve (CN II). Ptosis is usually related to stroke or paralysis in the eye area. A halo is seen in patients with glaucoma, lens opacities, and some drug toxicities.)

Which of the following is an outcome appropriate for a patient with hearing impairment? a. Provide a communication board or picture to assist teaching. b. Minimize background noise and close door. c. Stand in front of patient and explain procedure. d. Patient explains plan to accommodate hearing impairment.

D (Rationale: A, B, and C are interventions. The outcomes are what the goals are for the patient.)

Which of the following are interventions the nurse makes to prevent infections? Select all that apply. a. Immediately discontinue use of medication if an adverse drug reaction is suspected. b. Prevent infection of the blood from corrupted central lines by changing daily. c. Contact provider for sepsis treatment with positive urine dipstick for leukocyte esterase and/or nitrite. d. Assess for surgical site infections for 30-90 days after an operative procedure e. Screen patients for sepsis using evidence-based care and report all patients with sepsis.

D (Rationale: A. The provider and the pharmacist will consult on the seriousness of the reaction, whether to stop the medication and whether to add the medication to the patient's allergy list. B. Determine how long the patient has had the central line and whether its use is warranted. C. Signs and symptoms of urinary tract infection include fever (>38°C [104.8°F]), dysuria, urgency, suprapubic tenderness, frequency, costovertebral angle pain or tenderness, positive dipstick for leukocyte esterase and/or nitrite, pyuria, (urine specimen with white blood cell [WBC] count ≥10/mm3), microorganisms seen on Gram stain, and positive urine culture of microorganisms (Centers for Disease Control and Prevention [CDC], 2017). Determine how long the patient has had the urinary catheter and whether there is still a need for its use.)

Acute airway obstruction is a situation that should be a. reassessed during the next visit. b. evaluated within 8 hours. c. further assessed thoroughly. d. quickly assessed and treated

D (Rationale: Acute airway obstruction is a life-threatening emergency that requires immediate treatment.)

The nurse is assessing a patient who has been taking antibiotics for 10 days. Oral assessment is important because of the increased risk for which of the following? a. Fordyce granules b. Pharyngitis c. Anosmia d. Candida albicans

D (Rationale: Antibiotics alter the normal flora of the mouth and may cause overgrowth of the yeast that exists in the mouth, which is C. albicans.)

The nurse assesses whether the patient outcome "Patient drinks 1 liters every shift" has been met. This is called a. assessment. b. planning. c. implementation. d. evaluation.

D (Rationale: Assessment occurs throughout different parts of the nursing process. When the intent of the assessment is to determine whether outcomes are met, this is referred to as evaluation.)

A patient with benign breast condition is likely to a. develop breast cancer later in life. b. require hormone replacement therapy. c. be a teenager. d. have it resolve after menopause.

D (Rationale: BBD occurs most often in patients aged 30 to 55 years and decreases or resolves after menopause. It does not predispose someone to breast cancer and is not treated with hormone replacement therapy.)

The nurse is caring for a patient with a sudden onset of chest pain. Which assessment is highest priority? a. Auscultate heart sounds. b. Inspect the precordium. c. Percuss the left border. d. Obtain pulse and blood pressure.

D (Rationale: Blood pressure serves as an indicator of hemodynamic stability in this acute situation. Evaluating the pulse and blood pressure indicates if the patient has an effective pulse. Although heart sounds will be auscultated, the highest priority is identifying the consequences of chest pain and cardiac ischemia. Abnormal heart sounds may or may not reflect ischemia.)

Mrs. Johnson, a transcriptionist, reports pain and burning in her right hand. What assessment procedures should you perform next? a. Trendelenburg and drawer signs b. McMurray and Thomas tests c. Bulge test and ballottement d. Phalen and Tinel tests

D (Rationale: Both Phalen and Tinel signs are specific findings with carpal tunnel syndrome. Based on Mrs. Johnson's occupation, she is at risk for this problem. Bulge and ballottement tests look for effusion in the knee joint. The McMurray test assesses for meniscus tears in the knee. The Thomas test is used to identify flexion contracture of the hip. The Drawer test is for knee injury and the Trendelenberg test is for hip disease.)

A man had a motor vehicular accident and fractured his right ankle. He was transferred from the emergency department to the orthopedic nursing unit for further observation and possible surgery in the next 12 hours. What is the priority nursing assessment of the admitting orthopedic nurse? a. Temperature b. Capillary refill proximal to the injury of the right ankle c. Capillary refill distal to the injury of the left ankle d. Capillary refill distal to the injury of the right ankle

D (Rationale: Capillary refill is the priority nursing assessment to evaluate tissue perfusion for orthopedic trauma patients. Temperature is not a priority nursing assessment. Assessment of capillary refill should be distal to the injury and not proximal. The patient fractured his right ankle, and assessment of the left ankle is not the priority.)

You note that an adolescent has uneven shoulder height. To differentiate functional from structural scoliosis, you ask the patient to a. stand up straight while you check the height of the iliac crest. b. flex the elbow and pull against your resistance. c. shrug both shoulders while you provide resistance. d. bend forward at the waist while you palpate the spine.

D (Rationale: Checking the height of the iliac crest will provide information about scoliosis but will not differentiate functional from structural. With functional scoliosis, the spine straightens with bending. This problem usually is associated with uneven leg length.)

Upon inspection, the nurse sees flesh-colored lesions surrounding the anal area. These lesions most likely indicate a. hemorrhoids. b. herpes simplex virus 2. c. AIDS. condyloma acuminatum infection

D (Rationale: Condyloma present as fleshy white to gray-appearing lesions. These lesions can be individual or may cluster in groups.)

Which of the following best describes the instructions the nurse should give a patient when assessing the thyroid from the posterior approach? a. Please tilt your head back as far as possible. b. Please turn your head as far to the right as you can. c. Please bring your chin down toward your neck. d. Please tilt your head slightly down and to one side.

D (Rationale: During assessment of the thyroid, it is helpful for the patient to relax the sternocleidomastoid muscle by turning the head slightly and lowering it slightly toward the chin. This position makes it easier for the nurse to palpate each lobe of the thyroid.)

Health promotion for children should incorporate teaching about lifelong cardiovascular health, including which of the following? a. Information on good nutrition b. Information on the prevention of illnesses c. Information on exercise d. All of the above

D (Rationale: Exercise and good nutrition with maintenance of an appropriate weight are important for cardiovascular health. By preventing illnesses with immunizations, pregnant women are not exposed to viral illnesses (e.g., rubella and rubeola) that may injure the developing fetal heart. Hand hygiene and avoidance of sick people may help decrease the spread of bacteria that could potentially injure the heart valves.)

Adult patients may have variations in pulse rates with a. respirations. b. food intake. c. heat. d. exercise.

D (Rationale: Exercise will increase heart rate because of increased metabolic demands. Sinus arrhythmia, a variation in pulse with respiration, is common among children. The pulse rate varies with respiration, speeding up during inspiration and slowing down during expiration.)

After completing a history on a 45-year-old patient, the nurse suspects the patient may have uterine fibroids. What information might have led her to this conclusion? a. History of STIs b. History of multiple births c. Vaginal discharge d. Heavier than usual menstrual periods

D (Rationale: Fibroids are suspected when a patient presents with heavy menstrual flow, irregular bleeding, pelvic pressure, or all of these symptoms.)

A male patient presents to the clinic with a complaint of a hard, irregular, nontender mass on his chest under the areola. Upon examination, the nurse notes that the mass is immobile and suspects a. gynecomastia. b. benign lesion. c. Paget disease. d. carcinoma.

D (Rationale: Gynecomastia is non-inflammatory enlargement of male breast tissue. Paget disease may cause intraductal carcinoma, presenting with clear, yellow discharge and dry, scaling crusts that spread outward from the nipple to the areola.)

A patient has dyspnea, edema, weight gain, and liquid intake greater than output. These symptoms are consistent with which nursing diagnosis? a. Ineffective cardiac tissue perfusion b. Decreased cardiac output c. Impaired gas exchange d. Excess fluid volume

D (Rationale: Ineffective cardiac tissue perfusion describes the lack of blood being supplied to the myocardium and relates to cardiac ischemia and chest pain. Impaired gas exchange, a respiratory diagnosis, focuses on the exchange of oxygen and carbon dioxide at the alveolar level. Decreased cardiac output relates to CHF and reduced circulation. Dyspnea from fluid in the lungs and edema and weight gain from fluid accumulation in the body support the most accurate labeling of excess fluid volume.)

Which of the following is an example of inspection? a. Heart rate and rhythm regular b. Lungs clear c. Abdomen tympanic d. Skin pink

D (Rationale: Inspection involves visual information.)

A patient presents with a complaint of drooping of the eyelid on one side. This finding is documented as which of the following? a. Kernig sign b. Pharyngitis c. Thyroglossal cyst d. Ptosis

D (Rationale: Kernig sign is found with meningitis. Pharyngitis is inflamed and sore throat. A thyroglossal cyst is a birth defect mass found in the neck.)

Latex allergies a. always result in anaphylactic reactions and shock. b. can be reduced by moisturizing the hands after washing. c. cannot be caused by equipment such as a stethoscope. d. are more common in nurses and in frequently hospitalized patients.

D (Rationale: Latex allergies are more common in nurses and frequently hospitalized patients. They may result in anaphylactic or less severe reactions (e.g., difficulty breathing, itching, hives). The only way to avoid latex reactions is to avoid exposure to latex, which may be present in some stethoscopes, equipment, and stoppers of some medication vials.)

Which of the following scores for distance vision indicates the patient with the poorest vision? a. 200/20 b. 18/20 c. 24/20 d. 20/100

D (Rationale: Normal refractive index is 20/20. Visual acuity for distance vision is documented in reference to what a person with normal vision can see standing 6 m (20 ft) in front of the test (which is the numerator). The numerator is compared to what a person with normal visual acuity could read on that particular line (which is the denominator). Someone with 20/20 vision can read at 20 ft what a person with normal vision can read at 20 ft.)

Which of the following patients should not have a temperature measured orally? a. An 84-year-old woman with diarrhea b. A 30-year-old patient with an earache c. A 45-year-old man with chest pain d. A 62-year-old woman who has had oral surgery

D (Rationale: Oral temperature measurement is contraindicated in patients who have altered mental status, those who are mouth breathers, those who have had recent oral intake or who have recently smoked, and those who have recently undergone oral surgery.)

A 92-year-old woman with a history of COPD presents with increasing shortness of breath, decreased lung sounds in the bases, increased ankle edema, and 5-lb weight gain in 1 week. What is the most likely problem? a. Impaired gas exchange b. Ineffective airway clearance c. Activity intolerance d. Excess fluid volume

D (Rationale: Patients with chronic obstructive pulmonary disease (COPD) often retain fluid because of the increased workload of the heart that the disease imposes. Fluid accumulates in the bases and peripheral parts of the lungs, leading to increased shortness of breath and weight gain.)

What is the best time to assess the respiratory rate of a young child? a. While the child is crying b. While the child is playing in the playroom c. Immediately after taking the child's BP d. While the child is quietly sitting on the parent's lap

D (Rationale: Respirations are best determined while the child is sleeping or quietly awake. When a child is playing or upset, respirations may increase from activity or crying.)

You are inspecting the groin of an older adult man who lives in a long-term care facility. Which of the following is an expected finding that you will document? a. Pediculosis in hair distribution b. Hypospadias on the glans c. Yellow discharge from the meatus d. Smegma under the foreskin

D (Rationale: Smegma is a thin, white, cheesy substance that may normally be present under the foreskin. Pediculosis is infestation with lice. Hypospadias occurs when the urethral meatus is on the ventral side of the penis. Yellow discharge indicates an infection.)

A 3-year-old boy is brought to the emergency department with stridor, nasal flaring, intercostal and supraclavicular retractions, and respiratory rate of 40 breaths/min. What type of situation is this? a. Stable b. Acute c. Urgent d. Emergency

D (Rationale: Stridor indicates upper airway obstruction and is considered an emergency. Because it is accompanied in this case by retractions and tachypnea, a rapid response may be indicated.)

A patient reports swelling in her ankles. How would the nurse proceed with physical examination? a. Have the patient elevate her feet to better visualize her ankles. b. Measure her ankles at their widest point. c. Evaluate further for the brown hyperpigmentation associated with venous insufficiency. d. Press the fingers in the edematous area evaluating for a remaining indentation after the nurse removes his or her fingers.

D (Rationale: Swelling requires evaluation for pitting edema.)

If the great toe extends upward and the other toes fan out in response to stroking the lateral aspect of the sole of the foot, this is documented as which of the following? a. Hyporeflexia b. Normal plantar reflex c. Cushing response d. Babinski sign

D (Rationale: The Babinski sign indicates pathological hyperreflexia. A normal plantar reflex would result in toes curling downward to the same stimulus. The Cushing response refers to a pattern of changes in vital signs, not reflexes.)

Nursing assessment of trends in an unconscious patient's neurological status over time is best recorded on a. an admission assessment b. a PO c. a progress note d. a focused assessment flow sheet

D (Rationale: The admission assessment is usually performed just once, upon admission to a facility. If the patient is unconscious or the data are incomplete, the nurse adds data to the admission assessment after 24 hours. Nurses use the plan of care to identify outcomes and direct future care so that nursing care is consistent from shift to shift. Progress notes evaluate patient progress toward outcomes. A judgment is made about progressing or not progressing toward goals. Outcomes and interventions may be revised as needed, and a reassessment is made. The focused assessment would have information just on the neurological assessment so that the treatment team could identify changes in the patient's status quickly. Nurses can incorporate data and trends into a plan of care and progress note to show how the assessment is a basis for interventions.)

The nurse evaluates circulation, movement, and sensation on the right leg of a patient who was admitted with a tibia/fibula fracture. This type of assessment is considered a a. head-to-toe assessment. b. comprehensive assessment. c. emergency assessment. d. focused assessment.

D (Rationale: The assessment focuses on the patient's problem and is usually combined with a screening assessment for common hospital complications.)

The proper technique for correcting written documentation is to a. use correction fluid and write over the error. b. completely black out the error with a black marker. c. write over the error in darker ink. d. draw a line through the error and write the date, time, reason for error, and your initials.

D (Rationale: The legal technique for correcting an error is to place a single line through it, write the word "error," and initial it. In a court of law, the court needs to see the underlying data that were corrected. Also, the nurse must record the correct information.)

All of the following skin lesions may be papular except warts. a. acne. b. nevi. c. herpes d. zoster.

D (Rationale: The lesions of herpes zoster are vesicular, warts and nevi or moles are benign papules, and acne lesions include papules as well as pustules.)

Which of the following are usually done in an APRN assessment but not an RN assessment? a. History and risk factors b. Symptom analysis c. Inspection and palpation d. Otoscopic assessment

D (Rationale: The nursing roles are similar, but the nurse practitioner also has developed expertise with the otoscopic assessment.)

What percussion sound is heard over most of the abdomen? a. Resonance b. Hyperresonance c. Dullness d. Tympany

D (Rationale: The small intestine and colon, which are hollow organs, are predominant over most of the abdominal cavity. The result is tympany as the percussion sound.)

A child's head circumference is a measurement that should be obtained at every well-child visit until the child is 5 years old. a. True. This measurement is indicative of brain growth. b. False. One or two measurements are the standard of care. c. True. It will provide information on the child's readiness for kindergarten. d. False. The charts for head circumference norms end at 36 months of age.

D (Rationale: There are no reference points after 36 months; however, if concern is noted, continued measurements are appropriate.)

Your patient with a humerus fracture is stating pain of 5 on a 10-point scale. His hand is pale, cool, and swollen. His pain medication is ineffective, and he is at risk for impaired circulation. What action will the nurse take first? a. Reassess the pain in 30 minutes and contact the provider if unresolved. b. Give additional pain medication and reassess the pain in 30 minutes. c. Document the abnormal findings and give an extra dose of pain medication now. d. Contact the primary care provider and document the findings now.

D (Rationale: This situation represents an acute emergency for which the nurse should take immediate action. Nurses communicate the assessment using the SBAR technique. In addition, they document the findings in the chart. Nurses also note the interventions, such as pain medication, effectiveness, and assessment. They document that they contacted the primary care provider and the response. If the response was unacceptable, nurses may continue to call using the chain of command or may initiate a rapid response.)

While assessing the skin of a 24-year-old patient, the nurse notes decreased skin turgor. The nurse should further assess for signs and symptoms of a. hyperthyroidism. b. hypothyroidism. c. malnutrition. d. dehydration.

D (Rationale: When water is lost from subcutaneous tissues, the skin becomes less elastic. The result is "tenting," which results when the skin is pulled away from the body and released. This is a sign of possible dehydration.)

The nurse performs the first assessment on the hospitalized patient and documents it in the chart as the a. sporadic assessment. b. functional assessment. c. focused assessment. d. comprehensive assessment.

D (Rationale: when the patient is admitted to the hospital, a more comprehensive assessment gathers the patient history, subjective data, and objective data. Assessments follow that focus on the problems identified.)

Which sexually transmitted infection presents with painful red superficial vesicles along the penis or on the glans? a. Gonorrhea b. Chlamydia c. Syphilis d. Herpes simplex virus 2 (HSV-2)

D. (Rationale: Herpes presents with painful vesicles along the penis or on the glans.)

You are triaging infants who have presented to the emergency department on a Friday night. Which infant should you take in for treatment first? a. A 2-week-old infant whose mother reports, "She just won't stop crying. I'm so worried." The cry is medium pitch; temperature 37°C (99°F), HR 160 beats/min, RR 50 breaths/min; abdomen moves with each breath. b. A 6-week-old infant whose father reports, "He's vomited several times and he won't take his bottle." Temperature 36°C (96.8°F), HR 70 beats/min, RR 20 breaths/min. His lips are white. He is limp. c. A 5-month-old infant with a stuffy nose who has been unusually fussy and has had three loose stools in the past 8 hours. Temperature 37.6°C (99.8°F), HR 140 beats/min, RR 45 breaths/min while crying. d. An 8-month-old infant whose parents report he choked on a bean at dinner. The bean came out after five back pats. He turns blue around his mouth when he cries. Temperature 37°C (98.6°F), HR 130 beats/min, RR 30 breaths/min.

a. A 6-week-old infant whose father reports, "He's vomited several times and he won't take his bottle." Temperature 36°C (96.8°F), HR 70 beats/min, RR 20 breaths/min. His lips are white. He is limp.(Rationale: This infant's vital signs are low; he is pale and limp. All these signs are very worrisome. Typically, heart and respiratory rates increase when an infant is stressed. By the time they start to fall, the infant is decompensating. Because he is pale (white lips), it is difficult to tell if he is cyanotic. The nurse needs to check his oxygen saturation.)

The nurse is assessing a 2-month-old infant whose mother brought her to the emergency department because the baby wasn't eating well and she "just looks sick." Which of the following assessment findings is most worrisome? a. Stiff neck with an arched back b. Circumoral cyanosis noted when crying c. PMI not palpable, anterior fontanel bulges slightly when crying d. Temperature 36.4°C (97.5°F), heart rate (HR) 160 beats/min, respiratory rate (RR) 38 breaths/min

a. Stiff neck with an arched back (Rationale: A stiff neck and arched back describe opisthotonos, which occurs with meningeal irritation. Meningitis will need to be ruled out.)

Which of the following symptoms is NOT an indicator of preeclampsia? a. Uncontrolled vomiting b. Headache c. Epigastric pain d. Hyperreflexia

a. Uncontrolled vomiting (Rationale: Headache, epigastric pain, and hyperreflexia are typical symptoms of preeclampsia. Uncontrolled vomiting is the defining characteristic of hyperemesis gravidarum.)

A normal fetal heart rate as auscultated with a Doppler sonometer is a. 90 beats/min. b. 120 beats/min. c. 100 beats/min. d. 180 beats/min.

b. 120 beats/min (Rationale: Normal fetal heart rate is 110 to 160 beats/min, using any method (Doppler, electronic fetal monitor, or fetoscope).)

A mother brings her 6-month-old infant to the clinic for a routine evaluation. At birth, the term infant weighed 3.5 kg (7 lb 12 oz) and was 51 cm (20 in.) long. He now weighs 4.6 kg (10 lb 2 oz). Which assessments are most important for you to do next? a. Obtain a thorough obstetrical and neonatal history and say, "I'm very worried that the baby hasn't gained more weight. What are you feeding him?" b. Measure head and chest circumference and length, then plot current weight, length, and head and chest circumferences on standardized growth charts. c. Review the immunization history, administer the Denver II assessment, and ask the mother if she has noticed any unusual patterns or behaviors. d. Screen for domestic violence and focus on the neurological, cardiac, and abdominal portions of the physical examination.

b. Measure head and chest circumference and length, then plot current weight, length, and head and chest circumferences on standardized growth charts.(Rationale: First, the nurse needs to determine in what percentile the anthropometric measurements fall to compare findings with measurements from previous visits. It is important to establish the trend in the infant's physical growth pattern.)

The nurse is performing patient teaching about normal changes during late pregnancy. These include which of the following? a. Dark cloudy urine b. Waddling gait c. Vaginal bleeding d. Sudden edema

b. waddling gait (Rationale: Increased levels of relaxin loosen the cartilage between the pelvic bones, resulting in the characteristic "waddling" walk of the third trimester. This is a normal change during pregnancy. Dark cloudy urine is not normal and suggests infection or renal impairment. Significant vaginal bleeding (more than scant spotting) is never normal in pregnancy before the start of labor. Sudden edema is abnormal and may indicate preeclampsia.)

An infant has a new onset of rash but otherwise seems well. Which interview question is best when trying to pinpoint a possible cause? a. "Was there a prolonged NICU stay?" b. "What treatments have you given her for the rash?" c. "Has anything changed lately, such as shampoos, soaps, or laundry detergent?"

c. "Has anything changed lately, such as shampoos, soaps, or laundry detergent?"(Rationale: Because the baby is otherwise well, the condition may be allergic or irritant dermatitis. Asking about a change in shampoo, soap, or laundry detergent will focus the line of questioning toward trying to pinpoint any allergen or irritant. Although it is important to ask about treatments that have been given, this question is less likely to elicit information that will help determine the cause.)

Michelle's fundal height measures 28 cm (11 in.). You expect the gestational age to be a. 20 weeks. b. 14 weeks. c. 28 weeks. d. 30 weeks.

c. 28 weeks. (Rationale: From 20 weeks' gestation on, the fundal height should equal the gestational age in weeks.)

A patient comes into the clinic for a scheduled NST when the nurse notes that the FHR tracing is nonreactive. Which of the following actions would be appropriate for the nurse to do first? a. Document the findings. b. Notify the provider. c. Change the mother's position. d. Instruct the patient to return to the clinic in 1 week for reevaluation of the fetal heart rate.

c. Change the mother's position (Rationale: A reactive nonstress test is indicative of a healthy fetus. If the monitoring strip is nonreactive, the nurse may offer the mother a position change or a drink of cold water or juice to stimulate the fetus. The fetal response helps to distinguish a true nonreactive test from a normal fetal sleep cycle. If the position change is not effective, this is an indicator of fetal distress and should be reported to the provider immediately. Genuine fetal distress may indicate urgent delivery of the baby to prevent a poor outcome. Findings should be documented after the intervention is complete. It would be inappropriate to wait for 1 week to intervene or reevaluate the fetal heart rate, as there is the possibility of fetal distress.)

Which of the following conditions would be the highest priority to contact the health care provider about? a. Striae gravidarum b. Varicosities of the labia c. Contractions before 37 weeks d. Prominent Montgomery glands

c. Contractions before 37 weeks (Rationale: Contractions prior to 37 weeks are symptomatic of preterm labor and may lead to preterm birth and poor outcomes for the baby. Striae gravidarum, labial varicosities, and prominent Montgomery glands are all normal findings in pregnancy.)

Which of the following 6-month-old infants has the most markers for a possible genetic disorder? a. Has large ears, is in the 95th percentile for weight and height, babbles b. Has large scaly plaques on face and torso, red reflex is absent in one eye, posterior fontanelle has closed c. Has significant head lag, one ear is small and malformed, nipples are unusually close together d. Sits up alone, cranial sutures are palpable, back of the head is flat

c. Has significant head lag, one ear is small and malformed, nipples are unusually close together (Rationale: This is the only answer choice with three markers that point to a possible genetic disorder. Large ears on a baby whose growth is in the 95th percentile on the growth chart is not a strong sign. Large scaly plaques are associated with psoriasis. An absent red reflex in one eye is abnormal but not a strong marker of a genetic disorder. Cranial sutures should be palpable; a head that is flat in the back is most likely from positional plagiocephaly related to sleeping on the back.)

Michelle says that her last normal menstrual period was June 15. Using the Nägele rule, her EDD is a. September 8. b. March 8. c. March 22. d. January 22.

c. March 22. (Rationale: The Nägele rule states that to determine estimated date of delivery, subtract 3 months from the first day of the last menstrual period and add 7 days to the result.)

The nurse assesses for possible complications of pregnancy. Which of these prompts referral to a perinatal specialist? a. Gastric reflux b. Previous cesarean procedure c. Oligohydramnios d. Anemia

c. Oligohydraminos (Rationale: Oligohydramnios is an insufficient level of amniotic fluid, which can result in poor fetal prognosis and perinatal complications. Gastric reflux and anemia are common features of pregnancy, which can be managed by a nurse practitioner (NP), certified nurse midwife, or physician. A previous cesarean procedure does not increase risk significantly enough to demand the care of a specialist and can also be managed by an NP, certified nurse midwife (CNM), or MD.)

You are evaluating the growth pattern of a 5-month-old infant born at 27 weeks' gestation. Which of the following actions will yield the most accurate assessment of growth for this infant? a. Calculate how many kilocalories per day the infant is consuming, evaluate his bowel movement pattern, plot his measurements, and compare with the last two visits. b. Determine whether he has gained at least 2.2 kg (5 lb) since birth, because infants should gain 500 g to 1 kg (1 to 2 lb) per month in the first 6 months. c. Plot the weight and length on a standardized growth chart for a 7-week-old infant and compare with birth measurements and measurements on previous visits. d. Plot the weight and length on a standardized growth chart for a 12-week-old infant and compare with birth measurements and measurements on previous visits.

c. Plot the weight and length on a standardized growth chart for a 7-week-old infant and compare with birth measurements and measurements on previous visits. (Rationale: This baby was born 13 weeks prematurely. At 5 months, he is now 20 weeks old. Subtract 13 from 20 to get 7 weeks, which is his corrected age.)

A patient calls the provider's office to schedule an appointment because a home pregnancy test was positive. The nurse knows that the test identified the presence of which of the following in the urine? a. Estrogen b. Progesterone c. hCG d. Follicle-stimulating hormone

c. hCG (Rationale: After implantation, the outer layer of the developing embryo (trophoblast) produces human chorionic gonadotropin (hCG). Pregnancy tests (both urine and blood) measure levels of this hormone, whose presence validates the existence of a pregnancy and initiates a feedback loop that preserves the corpus luteum.)

You evaluate all the following children one morning in the clinic. Which should you refer for further assessment? a. A 6-week-old boy whose parents recently immigrated from Thailand; his head lags when pulled up by his arms; he has several dark spots that look like bruises on his lower back and buttocks. b. A 4-week-old African American girl whose liver margins are barely palpable along the right costal margin; her kidneys are easily palpable: her ears look "funny." c. A 4-month-old Caucasian boy with loud breath sounds throughout the lung fields; auscultation of the heart reveals a split S2. d. A 9-month-old Latina who is fussy; her tympanic membrane is pearly gray and moves during pneumatic otoscopy.

d. Baby is sleeping supine in her crib, no pillow, one blanket, bottle lying beside baby and a tiny dribble of milk at the corner of her mouth.(Rationale: Palpable kidneys mean they are enlarged. In addition, "funny-looking" ears could be another sign of kidney problems.)

Which of the following activities best facilitates anticipatory guidance? a. Becoming very proficient in interviewing and performing the physical examination b. Doing as much of the examination as possible with the infant in the parents' lap c. Recognizing and reporting signs of physical abuse and neglect d. Encouraging parents to make an appointment with the pediatrician before the baby is born

d. Encouraging parents to make an appointment with the pediatrician before the baby is born (Rationale: All the actions mentioned are good things to do; however, encouraging a prenatal visit to the pediatrician sets up the opportunity for parents to ask questions and for the pediatrician to help prepare the parents (anticipatory guidance) for the new baby.)

The nurse is caring for a patient who is admitted to the hospital with a possible ectopic pregnancy. Which of the following nursing actions is the priority? a. Monitoring daily weight b. Assessing for edema c. Monitoring the temperature d. Monitoring the blood pressure

d. Monitoring the blood pressure (Rationale: A significant drop in blood pressure is an indicator of hemorrhage caused by a ruptured ectopic pregnancy. Temperature, edema, and weight are important, but hemorrhage is the life-threatening concern.)

Which of the following infants has the most signs that point to possible abuse? a. History of a long NICU stay for extreme prematurity; does not respond to loud clapping b. Positive Ortolani and Barlow maneuver results; one leg looks shorter than the other c. Small baby with large areas of denuded skin on his face and torso d. When baby cries, mother says, "Shut up already." Baby has a foul odor and looks dirty.

d. When baby cries, mother says, "Shut up already." Baby has a foul odor and looks dirty. (Rationale: The mother's response indicates an inability or unwillingness to respond to the baby's cues. The foul odor and uncleanliness signify possible neglect. A careful physical examination, with the nurse looking for other signs of abuse, is in order.)


Kaugnay na mga set ng pag-aaral

el gran robo argentino sentences

View Set

Legal Concepts of the insurance Contract- chapter 2

View Set

ACCT 3230 - Chapter 14 - LearnSmart

View Set

CHOICE IN A WORLD OF SCARCITY QUIZ

View Set

Stress Management and Designing a Personal Fitness Plan

View Set

Ethics for the information age CH 1-10

View Set

Head and Neck Tumors/Parotid, Anatomy

View Set